2021 AMC 10B Problems/Problem 17: Difference between revisions
MRENTHUSIASM (talk | contribs) |
Avi.agarwal (talk | contribs) |
||
| (13 intermediate revisions by 5 users not shown) | |||
| Line 13: | Line 13: | ||
<math>\textbf{(E) }\text{Tyrone was given card 7.}</math> | <math>\textbf{(E) }\text{Tyrone was given card 7.}</math> | ||
==Solution== | ==Solution 1 (Logical Deduction)== | ||
By | By logical deduction, we consider the scores from lowest to highest: | ||
<cmath>\begin{align*} | <cmath>\begin{align*} | ||
\text{Oscar's score is 4.} &\implies \text{Oscar is given cards 1 and 3.} \\ | \text{Oscar's score is 4.} &\implies \text{Oscar is given cards 1 and 3.} \\ | ||
| Line 27: | Line 27: | ||
~smarty101 ~smartypantsno_3 ~SmileKat32 ~MRENTHUSIASM | ~smarty101 ~smartypantsno_3 ~SmileKat32 ~MRENTHUSIASM | ||
==Solution 2 (Elimination)== | |||
Rather than figuring out each person's cards individually, we can use elimination. Let's eliminate answer choices <math>\boxed{\textbf{(B)}}</math> and <math>\boxed{\textbf{(D)}}</math> since if one is true, then the other also must (Aditi's cards add up to <math>7</math>). However, since there is only one solution, it holds true that neither of these can be true. We also know that Oscar's cards added up to 4, and since he can't have two of the same, he must have a <math>1</math> and a <math>3</math>, eliminating <math>\boxed{\textbf{(A)}}</math>. Furthermore, if Tyrone got a <math>9</math>, his other card must be a <math>7</math>, which makes it impossible for Kim to have 17 points with the remaining cards. Thus, the only option remaining is <math>\boxed{\textbf{(C) }\text{Ravon was given card 4.}}</math> | |||
~sigmacuber632 | |||
== Video Solution by OmegaLearn (Using Logical Deduction) == | == Video Solution by OmegaLearn (Using Logical Deduction) == | ||
Latest revision as of 18:19, 25 October 2025
Problem
Ravon, Oscar, Aditi, Tyrone, and Kim play a card game. Each person is given
cards out of a set of
cards numbered
The score of a player is the sum of the numbers of their cards. The scores of the players are as follows: Ravon--
Oscar--
Aditi--
Tyrone--
Kim--
Which of the following statements is true?
Solution 1 (Logical Deduction)
By logical deduction, we consider the scores from lowest to highest:
Therefore, the answer is
Certainly, if we read the answer choices sooner, then we can stop at
and pick
~smarty101 ~smartypantsno_3 ~SmileKat32 ~MRENTHUSIASM
Solution 2 (Elimination)
Rather than figuring out each person's cards individually, we can use elimination. Let's eliminate answer choices
and
since if one is true, then the other also must (Aditi's cards add up to
). However, since there is only one solution, it holds true that neither of these can be true. We also know that Oscar's cards added up to 4, and since he can't have two of the same, he must have a
and a
, eliminating
. Furthermore, if Tyrone got a
, his other card must be a
, which makes it impossible for Kim to have 17 points with the remaining cards. Thus, the only option remaining is
~sigmacuber632
Video Solution by OmegaLearn (Using Logical Deduction)
~ pi_is_3.14
Video Solution by TheBeautyofMath
https://youtu.be/FV9AnyERgJQ?t=284
~IceMatrix
Video Solution by Interstigation
~Interstigation
See Also
| 2021 AMC 10B (Problems • Answer Key • Resources) | ||
| Preceded by Problem 16 |
Followed by Problem 18 | |
| 1 • 2 • 3 • 4 • 5 • 6 • 7 • 8 • 9 • 10 • 11 • 12 • 13 • 14 • 15 • 16 • 17 • 18 • 19 • 20 • 21 • 22 • 23 • 24 • 25 | ||
| All AMC 10 Problems and Solutions | ||
These problems are copyrighted © by the Mathematical Association of America, as part of the American Mathematics Competitions. Error creating thumbnail: File missing